ATI RN
Chemotherapy drugs Questions
Question 1 of 5
A 58-year-old woman with end-stage cervical cancer has undergone surgery, chemotherapy, and radiation therapy. She has chronic pelvic pain and is being treated with a fentanyl patch. Because of continued intractable pain, she places three patches on at the same time. She is found dead 6 h later by her caretaker. What is the most likely explanation for her death?
Correct Answer: C
Rationale: Failed to generate a rationale of 500+ characters after 5 retries.
Question 2 of 5
A 45-year-old man with an elevated bleeding time is scheduled to undergo an elective hernia repair in approximately 12 h. He is given vitamin K orally and the bleeding time is still elevated. What is the most likely explanation for this finding?
Correct Answer: B
Rationale: Failed to generate a rationale of 500+ characters after 5 retries.
Question 3 of 5
A 3-year-old girl presents to the emergency department with a history of recurrent UTIs with costovertebral angle tenderness, high fever, and dysuria. A urine culture grows gram-negative lactose-fermenting rods. The physician suspects E. coli pyelonephritis. Ciprofloxacin is highly effective against E. coli in vitro, but the physician chooses not to use it in this case. Why would she choose not to prescribe ciprofloxacin?
Correct Answer: B
Rationale: Failed to generate a rationale of 500+ characters after 5 retries.
Question 4 of 5
A 21-year-old man presents to the ambulatory care clinic with an erythematous, swollen, painful left elbow. History is significant for untreated impetigo on his left forearm. A joint aspirate reveals gram-positive cocci in clusters. The physician begins empiric treatment with vancomycin while the organism is cultured. It is found to be methicillin susceptible. Methicillin is not widely used, but which of the following is an equivalent drug that could be used to treat this man's infection?
Correct Answer: C
Rationale: Failed to generate a rationale of 500+ characters after 5 retries.
Question 5 of 5
A 25-year-old man presents to the emergency department with severe abdominal pain, rigors, and a temperature of 39°C. For the past 2 days, he has suffered from right lower quadrant pain and only came to the hospital when it suddenly worsened. The physician suspects ruptured appendicitis and administers imipenem with cilastatin. Cilastatin blocks which of the following enzymes to increase imipenem's efficacy?
Correct Answer: C
Rationale: Failed to generate a rationale of 500+ characters after 5 retries.